USMLE Physiology Step 1 MCQ

77
USMLE Physiology Step 1 MCQ There are 5 different classes of immunoglobulins. What part of the immunoglobulin determines it class: a. light chains b. Fab fraction c. Fc fraction d. the constant region of the heavy chain e. the variable region of the heavy chain f. reaction to antigen g. how fast the antibody moves in blood h. both light and heavy chains According to differences in their heavy chain constant domains, immunoglobulins are grouped into five classes or isotypes: IgG, IgA, IgM, IgD, and IgE. D Posted by sbmedex at 6:31 PM 0 comments Links to this post Labels: class , IgA , IgE , IgG , IgM , Immunoglobulins TUESDAY, FEBRUARY 17, 2009 USMLE MCQ Step 1 & 2- MCQ A 7 month old infant with multiple medical problems required intravenous access. The medical student asked the radiology resident to help place the long peripheral line in the left arm. Once the line was placed, a chest x ray was obtained. please see the attached link . The next step in the management of the infant is? a. fire the medical student b. fire the resident c. remove the line d. hide the x ray e. hide the infant f. leave the line alone

description

physiology mcqs for preparation of usmle step 1,

Transcript of USMLE Physiology Step 1 MCQ

Page 1: USMLE Physiology Step 1 MCQ

USMLE Physiology Step 1 MCQ

There are 5 different classes of immunoglobulins. What part of

the immunoglobulin determines it class:

a. light chains

b. Fab fraction

c. Fc fraction

d. the constant region of the heavy chain

e. the variable region of the heavy chain

f. reaction to antigen

g. how fast the antibody moves in blood

h. both light and heavy chains

According to differences in their heavy chain constant domains,

immunoglobulins are grouped into five classes or isotypes: IgG, IgA, IgM,

IgD, and IgE. D

Posted by sbmedex at 6:31 PM 0 comments Links to this post

Labels: class, IgA, IgE, IgG, IgM, Immunoglobulins

T U E S D A Y , F E B R U A R Y 1 7 , 2 0 0 9

USMLE MCQ Step 1 & 2- MCQ

A 7 month old infant with multiple medical problems required

intravenous access. The medical student asked the radiology

resident to help place the long peripheral line in the left arm.

Once the line was placed, a chest x ray was obtained. please see

the attached link. The next step in the management of the infant

is?

a. fire the medical student

b. fire the resident

c. remove the line

d. hide the x ray

e. hide the infant

f. leave the line alone

Page 2: USMLE Physiology Step 1 MCQ

g. put a new line in the right arm

h. pull the line until it is just at the tip of the heart

ANS: The chest x ray shows a PICC line placed in the arm and entering the

left axillary and subclavian vein. Then the line enters the heart

through the LEFT SUPERIOR VENA CAVA, goes through the coronary

sinus and comes out of the inferior vena cava. This is not abnormal. All of

us have one superior vena cava on the RIGHT. Some infants born with

congenital heart disorders also have a left superior vena cava. The left

superior vena cava then connects to the coronary sinus in the right

atrium. THE LEFT SUPERIOR VENA CAVA IS NORMAL. Sometimes it may

open into the left atrium and this is known as an unroofed coronary sinus

and acts like an atrial septal defect. In general, the shunting is minimal

and nothing needs to be done.

Posted by sbmedex at 8:27 PM 0 comments Links to this post

W E D N E S D A Y , F E B R U A R Y 1 1 , 2 0 0 9

USMLE step 1 & 2, Microbiology MCQ

A 5 yo presented to the ER with stridor and drooling in a child.

The mother says the child has not been able to eat and is crying.

He is febrile and irritable. He has no past medical history of any

significance, nor does he have any allergies. He is barely able to

open his mouth. A lateral head and neck x ray is obtained. Follow

this link for the x ray- ( a-5-yo-) What should not be done to this

patient?

a. give oxygen

b. start antibiotics

c. have tracheostomy set at bed side

d. oral examination and obtain swabs for culture

e. observe closely in ICU

Epiglottitis is a medical emergency that may result in death if not treated

quickly. The epiglottis is a small flap of tissue that is located at the tongue

base. It's function is to prevent food from going down the trachea during

Page 3: USMLE Physiology Step 1 MCQ

swallowing

When the epiglottitis is infected or inflamed it can quickly obstruct the

large ariways and prevent air from going down the trachea. The posterior

throat is tender and one is not able to swallow- thus most individuals

drool.

The infection often spreads to the surrounding areas. With continued

inflammation and swelling of epiglottis, complete blockage of the airway

may occur. Death is not unheard of. when a patient presents with such a

scenario, the last thing one should do is inspect the mouth. the slightest

irritation can lead to complete shutdown of the airways. all precautions for

an emergency airway must be made available at the patient's bedside.

Epiglottitis is an infection of children and almost never seen in adults.

Today, vaccination against Haemophilus influenzae type b (has

significantly decreased the incidence of epiglottitis in children.

Epiglottitis caused by Hib is most common in children aged 2-7 years but

for some unknown reason has not been reported among Navajo Indians

and Alaskan Eskimos.

X ray show a V Shaped air density and then no more air going into the

trachea. The rapid tapering of air entry (V sign) is often seen in children

who have epiglottitis. This tells you that the trachea is blocked from the

excess swelling and any manipulation of the mouth can immediately

cause respiratory arrest. Always have an emergency tracheostomy kit at

the bedside incase you need access to the airways.

Make no attempt at home to inspect the throat of a person suspected of

having epiglottitis.

Manipulation of the oral cavity may result in sudden airway closure. Even

oral intubation is considered dangerous and thus a tracehostomy set

should always be placed at the bedsite.

Page 4: USMLE Physiology Step 1 MCQ

Primary treatment of epiglottitis is relieving stress, giving oxygen, have a

quiet environment and close monitoring. IV fluids and antibiotics should

be administered.

For more pictorial based questions for the USMLE, step 1 and 2, write

to [email protected]

Posted by sbmedex at 6:33 AM 0 comments Links to this post

Labels: USMLE Microbiology MCQ

USMLE ECG Review MCQ

A 43 yo female with a dilated cardiomyopathy suddenly

developed this rhythm (see link). The patient has a BP of 50/10

and the pulse is non existent. You are on call to the Medical Floor

and the nurse asks you to do something quick. Please look at the

attached link and decide what the best treatment is?

a. beta blocker

b. quinidine

c. amiodarone

d. digoxin

Page 5: USMLE Physiology Step 1 MCQ

e. Defibrillation

Amiodarone is an anti arrhythmic drug used for various types of

tachyarrhythmias, both ventricular and supraventricular (atrial)

arrhythmias. it is a superb drug which can immediately reverse atrial

fibrillation and is also helpful for ventricular tachycardias. However, in any

patient with low blood pressure and a rhythm which is shown (VF), urgent

defibrillation is required. The first treatment of choice in a patient with

ventricular fibrillation is defibrillation at 360 joules. E

For more USMLE MCQs, 500 similar USMLE MCQs for $10

Posted by sbmedex at 5:39 AM 0 comments Links to this post

Labels: USMLE ECG

T U E S D A Y , F E B R U A R Y 1 0 , 2 0 0 9

USMLE Step 1 Review -Pharmacology

please select the one correct answer

1. A 19 yo with no symptoms had this ECG before a work out. He

did have abnormal palpitations a year ago but required no

therapy. He was not given any medical therapy but did try some

over the counter herbal medications. His ECG is shown here. hb-

11 . He most likely has?

a.first degree HB

b.atrial flutter

c.Normal sinus rhythm

d.atrial fibrillation

e.ventricular arrhythmia

f.Torsades de pointes

First degree heart block is fairly common. It can sometimes be found in

teenagers, young adults and in well-trained athletes. The condition may

be caused by rheumatic fever, some types of heart disease and by some

drugs. First-degree heart block causes no symptoms. When the PR

Page 6: USMLE Physiology Step 1 MCQ

interval is > 0.2 ms, the diagnosis of first degree HB is made. This type of

HB is fairly common in young teenagers and in athletes. It can be caused

by congenital heart disease and some drugs like digoxin. The disorder is

asymptomatic and requires no treatment. C

2. A 43 yo with mitral stenosis is found to have a dilated left

atrium. She is asymptomatic and had a BP of 90/50. You are

unable to feel her pulse. The chest x ray reveals a large heart. As

a precaution, your chief resident decides to place her on warfarin.

Of all the below ECG rhythms, she most likely has?

a. atrial flutter

b. premature atrial beats

c. first degree heart block

d. atrial fibrillation

e. ventricular tachycardia

f. sinus tachycardia

Atrial fibrillation is common with advancing age; however other causes

include uncontrolled hypertension, coronary disease, CHF, valvular heart

disease, acute pulmonary processes, hyperthyroidism, and acute alcohol

intoxication. Patients having undergone cardiac, pulmonary, or

esophageal surgery have a 20-40% postoperative incidence of atrial

fibrillation (AF). Patients with congenital heart disease are at higher risk

for developing AF. The most common arrhythmia seen in patients with

mitral stenosis is atrial fibrillation. The dilated atrium also has a tendency

to form a clot which frequently embolizes. D

3. The most common type of immune deficiency in children is?

a. AIDs

b. DiGeorge

c. Selective IgA

d. SCID

e. CGD

The most common congenital immune deficiency is selective IgA. There

Page 7: USMLE Physiology Step 1 MCQ

also exists selective IgM and IgG deficiencies, but they are less common.

IgA deficiency may be due to a failure of heavy-chain gene switching. C

4. Deficiency of the enzyme adenosine deaminase (in RBCs) is

seen in?

a. AIDs

b. SCIDs

c. DiGeorge

d. CGD

e. Jobs

Autosomal Recessive. Adenosine Deaminase deficiency leads to

accumulation of dATP. This leads to a decrease in DNA precursors

molecules. Severe deficiency in both humoral and cellular immunity

occurs due to impaired DNA synthesis. Bone marrow transplant may be

helpful in treatment. B

5. Fibrinolytic therapy can be used in all medical disorders

except:

a. a recent hemorrhagic brain injury

b. recent head trauma

c. closed head or facial injury

d. presence of a brain tumor

e. presence of a brain AV malformation or a hemangioma

f. presence of an aortic dissection

g. embolic occlusion of the femoral artery

h. DIC

i. Active GI bleed

Fibrinolytic therapy should never be used if there in any condition where

bleeding has occurred or can occur. The majority of embolic occlusions of

arteries are from blood clots. In such cases, fibrinolytic therapy can be

used to dissolve the blood clot. G

6. After starting a patient on a heparin drip for a pulmonary

Page 8: USMLE Physiology Step 1 MCQ

embolus, the earliest time to monitor the first set of PTT is:

a. 4-6 hrs

b. 12 hrs

c. 24 hrs

d. 1 hr

e. 4 hrs

After starting Heparin, the PTT is monitored every 4-6 hrs for the first 24

hours. When the PTT is therapeutic between 55-70 s, the PTT can be

monitored on a daily basis. A

7. The lytic agent which acts directly on plasminogen is:

a.Streptokinase

b.Urokinase

c.Hirudin

d.Warfarin

e.Heparin

f.Plavix

g.Ticlodipine

h.Aspirin

Urokinase is a serine protease which acts directly on plasminogen.

Urokinase is also found in physiological amounts in the blood and

prostate. It has the ability to directly act on plasminogen and breaking it

into plasmin. The activated plasmin then breakdowns the blood clot or

thrombin. A

8. A 49 year old female is admitted with pain her left calf. You

suspect she may have a DVT and order a Doppler ultrasound. The

test indicates that she has a 3 cm blood clot in the proximal

popliteal vein. You decide to start her on an anti coagulant.

Which is false about Heparin?

a. It is a mucopolysaccharide

Page 9: USMLE Physiology Step 1 MCQ

b. It is often administered two – three times daily

c. It binds to anti-thrombin 111 for its mode of action

d. It can be used as an oral dose when treating DVT

e. A PTT times should be monitored in patients receiving Heparin

f. It occurs naturally in the body

Heparin is a mucopolysaccharide, which is normally found in the basophils

and mast cells. It acts as an anti coagulant and helps prevent the

formation of blood clots. Unlike fibrinolytics, heparin does not break down

the already formed clots. Heparin is used to treat a variety of medical

disorders including acute myocardial infarction, atrial fibrillation, deep

vein thrombosis, pulmonary embolism; Heparin can never be used as an

oral agent and is ether administered subcutaneously or intravenously. Its

activity needs to be monitored by measuring PTT levels. D

9. The following patient is started on heparin. During the 3rd day,

you are asked which of the following is not a complication of

Heparin therapy.

a.Thrombocytopenia

b.osteoporosis

c.alopecia

d.urticaria

e.skin necrosis

Heparin is generally a safe drug but does have a few side effects. The

most serious is heparin induced thrombocytopenia (HIT) which presents

with occlusion of the arteries. The heparin should be stopped immediately

and the platelet count should be monitored. The condition can be

reversed and heparin use should be avoided. Other rare side effects of

heparin include osteoporosis, alopecia, uritcaria, skin necrosis commonly

occurs with Warfarin. E

10. Which of the anti coagulants has been associated with skin

necrosis?

Page 10: USMLE Physiology Step 1 MCQ

a. aspirin

b. Ticlodipine

c. Warfarin

d. Urokinase

e. streptokinase

Rarely, Warfarin has been known to cause a severe type of skin necrosis.

This adverse side effect generally tends to occur early after initiation of

treatment and has been linked to protein C or S deficiency. Protein C is an

innate anticoagulant that, like the procoagulant factors that Warfarin

inhibits, requires vitamin K-dependent carboxylation for its activity. Since

Warfarin initially decreases protein C levels faster than the coagulation

factors, it can paradoxically increase the blood's tendency to coagulate

when treatment is first begun. This is the reason why many patients are

fist given heparin prior to starting Warfarin. Occasionally, large blood clots

may form all over the body, esp. the extremities. C

Posted by sbmedex at 2:58 PM 0 comments Links to this post

Labels: USMLE Review Step 1

USMLE Step 1 Respiratory Pharmacology

Please choose one correct answer

1. The anti-asthmatic drug that can cause convulsions and

arrhythmias is?

a.Prednisone

b.Salmeterol

c.Zafirlukast

d.Theophylline

e.Ipratropium

Symptoms of toxicity of Theophylline include Abdominal pain, continuing

or severe; confusion or change in behavior; convulsions (seizures) ; dark

or bloody vomit; diarrhea; dizziness or lightheadedness; fast and/or

irregular heartbeat; nervousness or restlessness, continuing; trembling,

Page 11: USMLE Physiology Step 1 MCQ

continuing. D

2. The mode of action of the Methylxanthines is?

a.inhibit Choline esterase

b.inhibit Phosphodiesterase

c.stimulate c-GMP

d.stimulate Beta-1 receptors

e.inhibit MAO

Methylxanthines inhibit Phosphodiesterase and increased CAMP. These

agents increase the contraction of the diaphragm. The most common

adverse side effects of these agents is nausea, but cardiac arrhythmias,

convulsion may also result. Beta blockers are useful agents to prevent the

side effects of theophylline. B

3. The methylxathnine that has been used to treat intermittent

claudication is?

a. Pentoxifylline

b. aspirin

c. theopylline

d. warfarin

e. ticlodipine

Pentoxifylline is said to improve symptoms of intermittent claudication by

decreasing blood viscosity A

4. The longest acting B2 bronchodilator is?

a. Ipratropium

b. Albuterol

c. Salmeterol

d. Theophylline

Salmeterol is long acting beta 2 agonists. C

Page 12: USMLE Physiology Step 1 MCQ

5. The prophylactic agent of choice for treatment of asthma is?

a. Cromolyn acetate

b. Prednisone

c. Ipratropium

d. Salbutamol

e. Theophylline

Cromolyn and nedocromil are given as aerosols for asthma. These

prophylactic agents inhibit the release of mediators form the mast cells.

They have no bronchodilator action but are capable of preventing early

and late responses to antigens. A

6. Bronchodilators which are first line agents in COPD are?

a. Ipratropium

b. Prednisone

c. Salmeterol

d. Ephedrine

e. Zafirkulast

Ipratropium is an antimuscarinic agent and prevents bronchoconstriction.

It is used also in asthmatic and is an effective first line drug. A

7. Anti-asthmatic drug that inhibit the leukotrienes receptor is?

a.Zafirlukast

b.Salbutamol

c.Ipratropium

d.Prednisone

e.Epinephrine

Zafirkulast is an oral leukotriene receptor antagonist used for treating

asthma. Leukotrienes are a group of chemicals manufactured in the body

from arachidonic acid. Release of leukotrienes within the body, for

Page 13: USMLE Physiology Step 1 MCQ

example, by allergic reactions, promotes inflammation in many diseases

such as asthma, a disease in which inflammation occurs in the lungs.

Zafirlukast blocks the binding of leukotriene types D4 (LTD4), and E4

(LTE4). The drugs are orally active and have been shown to be effective in

preventing exercise and antigen asthmatic attacks. A

8. An asthmatic has been prescribed oral steroids for 2 weeks. He

later presents with a sore throat and pain on eating food. He

claims that his food does not taste right. The best treatment for

him is?

a.oral Nystatin

b.increase dose of steroids

c.decrease dose

d.start Acyclovir

e.start proton pump inhibitor

Oropharyngeal candidiasis is common with use of steroids. When steroids

are prescribed for any length of time, oral Nystatin should be prescribed.

A

9. Aspirin allergy is related to?

a.Arachidonic acid

b.Bradykinin

c.Leukotrienes

d.Histamine

e.Serotonin

Aspirin allergy results from the formation of leukotrienes. The leukotrienes

are known to cause bronchoconstriction. C

10. The slow releasing substance of anaphylaxis is?a. Histamine

b. Bradykinin

c. Serotonin

d. Leukotrienes

Page 14: USMLE Physiology Step 1 MCQ

e. Acetylcholine

The leukotrienes comprise the important mediators of

bronchoconstriction. D

Posted by sbmedex at 2:31 PM 0 comments Links to this post

Labels: USMLE Respiratory Pharmacology MCQs

USMLE Pharmacology- Heart Failure

Please select the one correct answer

1. For a 65 year old male with shortness of breath, diaphoresis

and basilar rales on auscultation, the least likely drug of choice

is:

a.Captopril

b.Furosemide

c.Atenolol

d.Digoxin

e.Minoxidil

Minoxidil is an alpha antagonist and has no role in the treatment of CHF. It

is used in the treatment of hypertension and has an odd side effect of

excess hair growth. E

2. A 59 year old female is placed on digoxin for atrial fibrillation.

He is seen in the medical clinic and you decide to add another

drug to his therapy. Which of the following drugs can increase the

toxicity of digoxin?

a.Potassium supplements

b.Vasotec

c.Vitamin D

d.Quinidine

e.Atenolol

Page 15: USMLE Physiology Step 1 MCQ

Quinidine is known to enhance the toxicity of digoxin by displacing digoxin

from the protein binding sites . D

3. A 71 year old female is admitted to the ICU with shortness of

breath and diaphoresis. She has been unable to lie down flat in

the bed. Auscultation reveals basilar rales and the presence of an

S3. She is unable to eat and you decide to start her on

intravenous therapy. The medication which can only be

administered intravenously is:

a.Digoxin

b.Captopril

c.Quinidine

d.Dobutamine

e.Amiodarone

Dobutamine is an inotropic agent with Beta 1 receptor agonist activity.

Besides increasing the force of contraction of the heart, it can also be a

vasodilator to the pulmonary arteries. D

4. A 71 year old has uncontrolled atrial fibrillation. He is

cardioverted and then started on oral digoxin therapy. He is later

seen in the ER with complaints of diarrhea, yellow vision and first

degree heart block. His condition has most likely been worsened

by?

a. hyperkalemia

b. hyponatremia

c. hypocalcemia

d. hypercalcemia

e. hypermagnesemia

Digoxin toxicity is worsened by hypokalemia, hypomagnesemia and

hypercalcemia. The toxicity is also worsened by Quinidine toxicity C

5. You have decided to admit a patient with a heart attack to the

Page 16: USMLE Physiology Step 1 MCQ

ICU. Four hours later, he is very dyspenic and the chest x ray

reveals an enlarged cardiac silhouette with Kerley B lines. Echo

reveals an ejection fraction of 20% and you start him on an agent

which inhibits the enzyme phosphodiesterase. The drug is?

a.Digoxin

b.Quinidine

c.Amrinone

d.Dobutamine

e.Amiodarone

Amrinone and milrinone are inotropic agents which act by inhibiting

phosphodiesterase. These agents increase the levels of cyclic AMP. C

6. A 71 year old with congestive heart failure is admitted to the

ICU. He is treated with diuretics, oxygen and a nitroglycerine

drip. Over the nest few days he improves and is transferred to

the floor. You decide to place him on a drug class that has been

shown to reduce mortality in patients with CHF. Your drug of

choice is

a.Beta blockers

b.Calcium channel blockers

c.Diuretics

d.ACE inhibitors

e.Phosphodiesterase inhibitors

ACE Inhibitors have been shown to reduce mortality and morbidity in CHF.

These agents reduce aldosterone secretion, salt and water retention and

decrease afterload. These agents are now considered first line drugs for

CHF D

7. Which of the following is false about digitalis?

a. increased PR interval is an early effect of digoxin

b. increase automaticity is caused by intracellular calcium overload

Page 17: USMLE Physiology Step 1 MCQ

c. decrease in conduction occurs at the AV node

d. the electrical effects of digoxin are mediated by the sympathetic

system

e. premature ventricular beats can occur with digoxin

The major electrical effects of digoxin are mediated by a parasympathetic

response which causes the slowing of the heart. These effects can be

reversed by Atropine. The basic effect of digoxin is to slow the ventricular

rate. D

8. Digoxin is used for all the below conditions EXCEPT:

a.atrial flutter

b.atrial fibrillation

c.CHF

d.ventricular tachycardia

Digoxin is the classic drug for CHF. However, it is now being replaced with

better and safer drugs. The drug has a tendency to cause toxicity which

can be aggravated by the use of diuretics and Qunidine. Digoxin has also

been used in the treatment of atrial fibrillation and flutter D

9. Which of the following statements is false about class 1

antiarrhythmic agents?

a.Quinidine can be used to treat both atrial and ventricular arrhythmias

b.Procainamide can cause a lupus like syndrome

c.Cinchonism can occur with the prolonged use of Quinidine

d.orally administered Lidocaine is effective for ventricular arrhythmias

e.Quinidine worsens digoxin toxicity

Lidocaine is useful for treatment of ventricular arrhythmias; especially

those occurring after an MI. Lidocaine can be administered intravenously,

intramuscular and even placed down the oral endotracheal tube. It is

never given orally as it is rapidly hydrolyzed. D

Page 18: USMLE Physiology Step 1 MCQ

10. The toxicity of class 1 antiarrhythmic agents can be treated

with?

a.Lidocaine

b.Digoxin

c.Sodium lactate

d.diuretics

e.steroids

All class 1 antiarrhythmic agents have the potential to cause arrhythmias.

Torsades can occur with quinidine. Hyperkalemia can worsen the toxicity

of these agents. Treatment of overdose is with sodium lactate to reverse

the hypotension and arrhythmias. C

Posted by sbmedex at 2:25 PM 0 comments Links to this post

Labels: USMLE Heart Failure MCQs

USMLE Step 1 Pharmacology- Anti Arrhythmics

Please Select one correct answer.

1. A 55 year old previously healthy male has been complaining of

chest palpitations for the past 3 months. He is seen in the

cardiology clinic. After a quick physical exam, the ECG reveals

that he has atrial fibrillation. Five minutes later, you noticed that

the patient is not responsive and has a Blood Pressure of 60/20.

The next step in his management is?

a. Amiodarone

b. Quinidine

c. Lidocaine

d. Cardioversion

e. Digoxin

In any hypotensive patient with an arrhythmia, Cardioversion should be

the first step. D

2. Which of the following is false about Amiodarone?

Page 19: USMLE Physiology Step 1 MCQ

a. is used for both atrial and ventricular arrhythmias

b. blocks sodium, calcium and potassium channels

c. can cause corneal deposits

d. pulmonary fibrosis is common

e. has a very short half life

Amiodarone is very effective for all arrhythmias but is limited in its used

by its toxicity. It acts by blocking the sodium calcium and potassium

channels. It has a number of toxicities including pulmonary fibrosis,

corneal deposits, hypothyroidism and tremor. The drug has a very long

half life. E

3. Which of the following is false about beta blockers?

a. Metoprolol is useful in the treatment of patients after an MI

b. Metoprolol can depress the cardiac output

c. Beta blockers reduce arrhythmias

d. Timolol can be used to treat glaucoma

e. oral Esmolol is effective for outpatient treatment of hypertension

Esmolol is a very short acting beta blocker for intravenous administration

of atrial arrhythmias and hypertension. The majority of beta blockers are

commonly used as prophylactic agents in patients who have had an MI.

these agents can also decrease atrial arrhythmias. Use should be

cautioned in patients with poor ejection fraction as these agents are

negatively inotropic. E

4. Which of the following is false about Adenosine?

a. Adenosine is a normal component of body tissues

b. the drug is effective is treating atrial arrhythmias

c. it has a very short duration of action

d. hypotension can occur

e. it depolarizes the AV node to decrease the nodal arrhythmias

Page 20: USMLE Physiology Step 1 MCQ

Adenosine is normal component of the body and when given

intravenously it is very effective for abolishing nodal arrhythmias. It acts

by hyperpolarizing the tissues. It has a very short duration of action and

can cause flushing and hypotension. E

5. Which of the following is false about calcium channel blockers?

a. can be used to treat hypertension and arrhythmias

b. block L calcium type channels

c. PR interval is typically shortened by these agents

d. Effective in patients with congestive heart failure

e. can be used to treat coronary artery spasm

Calcium channel blockers are widely used in the treatment of atrial

arrhythmias, hypertension and in the treatment of Prinzmetal’s angina.

These agents act by blocking the type calcium channels. They are

negative inotropes and should be used with caution in patients with heart

failure. D

6. Digoxin induced arrhythmias can be decreased by?

a.Calcium

b.Sodium

c.Magnesium

d.Uranium

e.Chromium

Magnesium is now felt to be an important ion in the genesis of

arrhythmias induced by digoxin. Magnesium is also used to treat

Torsades. Occasionally potassium depletion may not be reversed until the

magnesium deficiency has first been corrected. C

7. The beta blocker which is classified as a class 3 anti-arrhythmic

agent is?

a.Metoprolol

Page 21: USMLE Physiology Step 1 MCQ

b.Atenolol

c.Esmolol

d.Sotalol

e.Pindolol

Sotalol is in the same anti arrhythmic class drug as Amiodarone. It is used

in the treatment of ventricular arrhythmias. D

8. The major mechanism of class 3 anti arrhythmic agents is?

a.inhibition of Sodium influx

b.inhibition of l type Calcium channels

c.reducing outward phase of Potassium current

d.inhibiting Na K ATPase

e.inhibiting Phosphodiesterase

The hallmark of class 3 drugs is prolongation of the action potential

duration. The action potential prolongation is caused by blockade of the

potassium channels that are responsible for repolarization of the action

potential C

9. The agent which can sometimes be used to treat digoxin

induced arrhythmias is?

a.Quinidine

b.Amiodarone

c.Phenytoin

d.Sotalol

e.Fleicanide

Phenytoin is an anticonvulsant and is sometime used to treat digoxin

induced arrhythmias. C

10. The diuretic used in the treatment of mountain sickness is?

a. Acetozolamide

Page 22: USMLE Physiology Step 1 MCQ

b. Furosemide

c. Thiazide

d. Spironolactone

e. Mannitol

Acetozolamide is used principally in veterinary medicine for its effects on

aqueous humor production in the treatment of glaucoma. It has also been

used for its diuretic action and in the treatment of metabolic alkalosis. In

humans, the drug has been used as adjunctive therapy for epilepsy and

for acute high-altitude sickness. A

Posted by sbmedex at 12:44 PM 0 comments Links to this post

Labels: USMLE Anti Arrhythmics MCQs

USMLE Step 1- Surgical Anatomy MCQs

Please Select the one correct answer

1. You are in the anatomy laboratory and dissecting the heart and

lung. Suddenly you wonder what the first branch of the aorta is?

a. right brachiocephalic artery

b. left common carotid

c. left subclavian artery

d. right coronary artery

e. right common carotid artery

The right coronary is the first branch of the aorta. D

2. A patient has been involved in a MVA. He presents to the

emergency room with severe pain in his right chest. The chest x

ray reveals numerous rib fractures and pulmonary contusion.

Which finding on the physical examination would make you

suspect that would make you suspect flail chest is?

a. difficulty breathing

b. increased stridor

Page 23: USMLE Physiology Step 1 MCQ

c. paradoxical breathing

d. low oxygen saturation

e. pain

f. hyperventilation

Paradoxical breathing is a sin que non of flail chest. These patients need

arterial blood gas monitoring, pain control and pulmonary care. C

3. The main reason a tracheostomy is not performed on the first

cartilage ring is because:

a. trachea is too narrowb

b. a high chance of subglottic stenosis

c. a high chance of tracheo-innominate artery fistula

d. inability to access it

e. technically impossible

f. chance of damaging esophagus

Tracheostomy performed in the first cartilage ring can result in subglottic

stenosis, which is almost impossible to repair. B

4. The most common mass in the anterior mediastinum in an

adult is?

a. thymoma

b. thyroid

c. teratoma

d. lymphoma

e. leukemia

f. seminoma

Thymoma is the most common mass of the anterior mediastinum. A

5. The left lateral border and apex of the cardiac silhouette is

made up by the

a. right atrium

b. right ventricle

Page 24: USMLE Physiology Step 1 MCQ

c. left atrium

d. left ventricle

The left ventricle makes up the major border of the left cardiac silhouette.

D

6. An angiogram reveals stenosis of the right innominate artery.

The surgeon asks you to get the patient ready for surgery. You

tell him that your anatomy knowledge is excellent. The artery can

best be exposed by a?

a. right thoracotomy

b. right supraclavicular incision

c. median sternotomy

d. neck incision

Innominate artery is best exposed by a median sternotomy. C

7. A patient with a traumatic aortic rupture undergoes aortic

repair. The patient presents with hoarseness after surgery. The

most likely cause of hoarseness is?

a. postoperative viral infection

b. damage to phrenic nerve

c. trauma to recurrent laryngeal nerve

d. injury to sympathetic chain

e. damage to vocal cords during intubation

Hoarseness is due to injury to the recurrent laryngeal nerve. The recurrent

nerve runs in between the left common carotid and the left subclavian

and can easily be injured by placing a clamp in that region. C

8. A 35 year old male is involved in a head on motor vehicle

accident. On arrival to the emergency room, he has severe

bruises to his chest but is otherwise stable. The chest X ray

reveals a widen mediastinum. An angiogram is ordered which

Page 25: USMLE Physiology Step 1 MCQ

reveals a traumatic aortic rupture. The most common site of

aortic rupture after severe blunt trauma to the chest is ?

a. ascending aorta

b. abdominal aorta

c. aortic arch

d. descending aorta distal to left subclavian artery

e. femoral artery

f. left common carotid artery

Following blunt trauma, the descending aorta just distal to the left

subclavian artery is most prone to injury. A chest x ray will reveal a widen

mediastinum and a CT scan is required for diagnosis. D

9. A new born is seen in the infectious disease clinic due to

repeated Candida infections. Blood work reveals that he has very

low levels of calcium and has suffered from numerous tetany

spells. He most likely has:

a. AIDs

b. SCIDs

c. DiGeorge

d. CGD

e. Jobs

f. renal failure

T-Cell deficiency from no thymus. Hypocalcemic tetany from primary

parathyroid deficiency. C

10. Failure to develop the 3 and 4th pharyngeal pouches are

linked to what immune disorder?

a. AIDs

b. SCIDs

c. DiGeorge

d. CGD

Page 26: USMLE Physiology Step 1 MCQ

e. Job’s

Failure of development of the 3rd and 4th Pharyngeal Pouches leads to

agenesis of the thymus and parathyroid glands. C

For a complete set of USMLE Review Questions, write to

[email protected]

Posted by sbmedex at 5:34 AM 0 comments Links to this post

Labels: USMLE Anatomy MCQs

M O N D A Y , F E B R U A R Y 9 , 2 0 0 9

USMLE Trauma MCQs

1. A 71 year old is admitted to the CCU and needs invasive

monitoring. It is decided to place a swan Ganz catheter and

measure the pulmonary catheter wedge pressure. This will allow

one to assess the pressure in the?

a. Right atrium

b. Left atrium

c. Left ventricle

d. Right ventricle

e. Aorta

f. Inferior vena cava

Pulmonary capillary wedge pressure (PCWP) provides an indirect estimate

of left atrial pressure (LAP). Although left ventricular pressure can be

directly measured by placing a catheter into the left ventricle by feeding it

through a peripheral artery, into the aorta, and then into the ventricle, it is

not possible to place the catheter into the left atrium unless there was an

atrial septal defect present. The swan Ganz catheter is usually inserted at

the bedside under sterile conditions. It is frequently used to monitor

patients in the CCU and ICU. B

2. A 64 year old male in the ICU needs quick IV access. The

resident decides to insert a central venous line. Which is not a

Page 27: USMLE Physiology Step 1 MCQ

complication of right internal jugular puncture?

a. pneumothorax

b. hemomediastinum

c. thoracic duct injury

d. tracheal puncture

e. phrenic nerve injury

f. carotid artery dissection

g. injury to esophagus

Central line insertion can be associated with a number of complications.

However, the benefits derived from their benefits far over-ride the risks of

potential complications. The most common complication of central line

insertion is pneumothorax. There is a belief that the incidence of

pneumothorax is higher if the central line is attempted via the subclavian

vein. Many physicians utilize ultrasound to help identify the neck veins

before cannulation. The reported incidence of pneumothorax is about 1%

but this is under reported.

Other complications of central lines include infection chiefly from

Staphylococcus aureus and Staphylococcus epidermidis. The risk of

infection are high when the individual is diabetic, immunocompromised

and obese. When a central line infection is suspected, the line should be

removed and the tip sent for culture. If the individual is febrile, blood

cultures should be obtained. No new line should be re inserted until the

cultures are negative or the fever has subsided.

Other rare complications of central lines include injury to the carotid

artery, air embolism, arrhythmias, pericardial tamponade and pain. G

3. A 45 year old obese female undergoes a laparotomy. The best

way to manage her post operative pain is?

a. over the counter pain pill

b. pain medication PRN

c. patient controlled analgesia

Page 28: USMLE Physiology Step 1 MCQ

d. Epidural anesthesia

e. acupuncture

Patient controlled analgesia is best for most individuals esp. children and

obese individuals. C

4. A 57 year old has been told that he has a mass behind the left

knee. Ultrasound reveals that it is a vascular mass. In general,

the most common presentation of a popliteal artery aneurysm

begins with?

a. pain

b. mass behind knee

c. decreased pulses

d. claudication

e. sudden onset of left cold leg

Popliteal artery aneurysms are the most common peripheral arterial

aneurysms and the second most common aneurysm after abdominal

aortic aneurysms. In the past, popliteal aneurysms have been associated

with a high rate of limb loss. When the aneurysm ruptures, there is a

sudden loss of blood supply to the distal leg. In many cases, embolization

to the distal blood vessels of the foot is also common.

Because of this awareness, all individuals with either asymptomatic or

symptomatic aneurysms of ≥ 2 cm in diameter should be considered for

elective pass surgery. While surgery was the standard of care in the past,

this has no changed with the availability of endovascular therapy.

Endovascular stenting is also an excellent option in the presence of a

ruptured aneurysm. Once a stent has been placed it is important that the

patient be placed in anticoagulation medications like ticlodipine. E

5. A 35 yo female has just delivered a baby and 2 days later is

found to have a swollen left leg. Ultrasound shows a deep vein

thrombus just beneath the popliteal vein. The next step in her

management is?

Page 29: USMLE Physiology Step 1 MCQ

a. coumadin

b. heparin loading then infusion

c. low molecular weight heparin

d. Aspirin

e. Green field filter

The primary aim for the treatment of DVT is to prevent pulmonary

embolism, reduce morbidity, and prevent or minimize the risk of

developing the postphlebitic syndrome. Anticoagulation is the mainstay of

the initial treatment for DVT. One may use either unfractionated heparin

or low molecular weight heparins to treat the DVT. Heparin does not lyse

the thrombus but prevents extension of thrombus. Heparin has been

shown to significantly reduce (but not eliminate) the incidence of fatal and

nonfatal pulmonary emboli as well as recurrent thrombosis. The chief

reason for the continuing risk of PE is mainly because heparin does not

affect the preexisting nonadherent thrombus. Heparin does not affect the

size of existing thrombus nor does it have any intrinsic thrombolytic

activity.

In less than 10% of patients is heparin therapy associated with complete

lysis. Heparin therapy has little effect on the risk of developing the

postphlebitic syndrome. The deep vein thrombus usually causes

destruction of the valves which almost always results in a high incidence

of chronic venous insufficiency and postphlebitic syndrome.

Warfarin therapy is usually started 3-5 days after heparin. The INR is

maintained anywhere between 2-2.5. Heparin must be overlapped with

oral warfarin because of the initial transient hypercoagulable state

induced by warfarin. This effect is related to the differential half-lives of

protein C, protein S, and the vitamin K–dependent clotting factors II, VII,

IX, and X. Long-term anticoagulation is definitely indicated for patients

with recurrent venous thrombosis and/or persistent or irreversible risk

factors. B

6. A 20 year old was involved in a motor vehicle accident. He had

Page 30: USMLE Physiology Step 1 MCQ

severe blunt chest trauma to the chest. In the ER he received 8

units of packed red blood cells. Soon after he became short of

breath and his PO2 was 55 on 100% oxygen. His chest x ray

revealed a complete white out and he had a Central venous

pressure of 11. At 48 hours after the injury, the nature of his lung

injury was most likely due to?

a. pulmonary contusion

b. ARDs

c. Pneumonia

d. PE

e. heart failure

f. lung fibrosis

Bruising or contusion of the lug is common after chest trauma. The

condition results in lung injury which is often marked by fluid and blood

accumulation in the lung parenchyma. This results in difficulty in

oxygenation and hyperventilation. In lung contusion the lung tissue

structure is intact and there is no frank tear or rupture. Pulmonary lung

contusion rarely occurs in isolation and is often associated with rib

fractures or soft tissue injury. The typical signs of lung contusion include

chest pain, fever, tachypnea and hemoptysis. The chest x ray may reveal

signs of a lung contusion, but a CT scan is definitive. A

7. A 43 yo is involved in a car accident and breaks both femurs.

There is concern that he may develop fat embolism syndrome.

The radiographic findings of fat embolus are most typically

evidenced at?

a. 0-12 hrs

b. 12-24 hrs

c. 24-48 hrs

d. 48-72 hrs

There is no radiological feature which is diagnostic of fat embolism

syndrome. Infact in the first few hours after injury, the chest x ray may be

Page 31: USMLE Physiology Step 1 MCQ

completely normal. Over the next 48 hours, some patients may develop

bilateral fluffy shadows which are associated with a decline in the

respiratory status. Some individuals may develop diffuse or patchy air

space consolidation, due to edema or alveolar hemorrhage; this is most

prominent in the periphery and bases. Radiological studies like the

Ventilation/perfusion scans may demonstrate a mottled pattern of sub-

segmental perfusion defects with a normal ventilator pattern. In most

cases, the earliest features on x ray may be seen around 48-72 hours.

CT may identify ill defined areas of centrilobular and sub pleural nodules

representing alveolar edema, microhemorrhage, and inflammatory

response secondary to ischemia and cytotoxic emboli may be seen. MRI of

the brain may reveal high-intensity T2 signals; this correlates with the

degree of neurological impairment found clinically.

Fat particles are not present in more than 50% of individuals with fat

embolism syndrome. There are some who advocate the use of

bronchoscopy with bronchoalveolar lavage to detect fat droplets in

alveolar macrophages as a means to diagnose fat embolism has been

described in trauma patients and patients with the acute chest syndrome

of sickle cell disease. However, diagnostic criteria vary and the sensitivity

and specificity are unknown. D

Posted by sbmedex at 8:38 PM 0 comments Links to this post

Labels: USMLE Trauma MCQs

USMLE Step 1 Review No 2

Please select one correct answer

1.A 67 year old male presents with pain and a throbbing right

sided headache. Examination reveals a tender temporal artery

and a biopsy is immediately done. The classic findings on

pathology will reveal?

a. granulomas in internal elastic lamina

b. granulomas in the adventitia

Page 32: USMLE Physiology Step 1 MCQ

c. granulomas in endothelium

d. hemorrhagic infarct

e. liquefactive necrosis

f. caseating granulomas

g. inclusion bodies

Histopathology in temporal arteritis will reveal an inflammatory infiltrate

surrounding a fragmented internal elastic lamina within the media of an

vessel wall. Predominantly mononuclear cells with giant cell formation are

seen, indicating the presence of a chronic inflammatory process.

Temporal arteritis is often known as giant cell arteritis and is a disorder of

the medium sized arteries, esp in the head and neck area. the temporal

artery is a branch of the external carotid artery and is often involved in

the disease process.

The disorder may occur in association with polymyalgia rheumatica in

about 25% of cases, which is characterized by the sudden onset of muscle

pain, fever and fatigue. the disorder usually occurs in the elderly. When

the temporal artery biopsy is positive for inflammation, the patient needs

to be urgently treated with corticosteroids to prevent blindness. A

2. A 4 year old child is seen in the hematology clinic with a large

discolored lesion on his trunk and left thigh. The mother says that

he has had the lesion since birth but it has grown over the past 3

months. She denies any other problems. The blood work reveals a

PT of 12.3, PTT of 38, CBC of 5, WBC of 11.2 and platelets of 25.

Examination reveals a large hemangioma which is non tender and

there is no evidence of bleeding. The most likely diagnosis is?

a. hemophilia

b. von willebrands disease

c. Kassebach merit syndrome

d. cherry red spot

e. Mongolian spot

In 1940, Kassebach and Merritt described a male infant with a discolored

raised lesion on the extremity which rapidly grew and invaded the entire

Page 33: USMLE Physiology Step 1 MCQ

left leg, scrotum, abdomen, and thorax. The infant also had a

consumptive thrombocytopenia resulting in low platelets. This disorder is

now known as Kassebach-Merritt syndrome. The disorder is known to be

associated with Kaposi hemangioendothelioma and thrombocytopenia . it

is believed that the hemangioma triggers a cycle of activation of the

coagulation cascade followed by the consumption of the clotting factors

and platelets. the activation of platelets and the release of the mediators

also causes growth of the vascular defect. C

3. A 22 yo has sea sickness and asks you for some medication.

You prescribe her a transdermal patch of?

a. Hismanal

b. chlorpromazine

c. metoclopramide

d. scopolamine

e. atropine

Scopolamine is a decent anti emetic agent and also prevents dizziness.

Sometimes is has also been used to treat abdominal cramps. In the past it

was frequently used as pre anesthetic agent and for prevention of motion

sickness. Today. Scopolamine is available as a patch which can be placed

behind the ear. It is useful in preventing travel and sea sickness. Many

scuba divers use scopolamine patches. There is evidence that

scopolamine may also have anti depressant effects. Individuals who have

glaucoma should be careful with scopolamine as it can precipitate attacks.

D

4. In which of the following cancer is the myc gene amplified?

a. small cell lung cancer

b. squamous cell of skin

c. actinic keratosis

d. carcinoid of small bowel

e. melanoma

Page 34: USMLE Physiology Step 1 MCQ

The myc protein acts like a transcription factor and is known to control

several other genes. Mutations the myc gene have been founding several

cancers including lung, Burkett’s and B cell leukemia. The myc oncogenes

are known to become activated by may become activated by either an

arrangement or amplification process. Breakage and re sealing of the

chromosomes is part of the process and may affect many genes.

Translocation of gene is a process of rearrangement and can occur

between chromosomes, such as between chromosome 8 and 14. This

arrangement results in expression of the myc gene and eventually into a

B cell lymphoma. A

5. A 15 year old is seen in the hematology clinic with complaints

of fatigue. Examination reveals mild jaundice and splenomegaly.

Blood work reveals a CBC of 2.9, Hct of 22 and Hb of 8. The mean

corpuscular hemoglobin concentration is markedly increased. The

most likely cause is?

a. sickle cell anemia

b. Thalassemia

c. hereditary spherocytosis

d. iron deficiency anemia

e. macrocytic anemia

An increased MCHC obtained from an electronic cell counter is a

characteristic feature of red cells in HS. MCHC values greater than the

upper limit of normal (35-36%) are common. This increased MCHC is a

result of mild cellular dehydration. The mean cell volume (MCV) in

patients with HS actually is low. This relatively low MCV may reflect

membrane loss and cell dehydration.

Hereditary spherocytosis is a genetic disorder with autosomal dominance,

and is common in individuals from Northern Europe and Japan. Today,

estimates indicate that at least 25% of cases are due to spontaneous

mutations. With an autosomal dominant trait, there is chance that an

individual has a 50% chance of passing the disorder onto his/her offspring-

assuming that the partner does not carry the spontaneously mutated

Page 35: USMLE Physiology Step 1 MCQ

gene. Hereditary spherocytosis causes defects in the genes that code for

proteins such as spectrin and ankyrin. These proteins are involved in the

formation and stabilization of red cell membranes. These proteins play a

major role in the maintenance of the shape of the RBC. In HS, it is ankyrin

which appears to be defective. The red cells formed are abnormal and

removed by the spleen. C

6. A 16 year old is admitted to the hematology clinic with

complaints of fatigue and joint pains. He has had RUQ pain on

numerous occasions which has subsided with pain medications.

Blood work reveals a hypochromic mild anemia and a blood smear

reveals intraerythrocytic crystals. Laboratory studies reveal that

these intraerythrocytic crystals disappear in the absence of

oxygen and appear when a slat solution (e.g. vinegar) is added.

He may have?

a. Thalassemia

b. Hemophilia

c. Von Willebrand disease

d. Idiopathic thrombocytopenic purpura

e. Hemoglobin C

f. Sickle cell anemia

g. Leukemia

The hemoglobin C cell forms circulating intraerythrocytic crystals

(tactoids) in the oxy state and has reduced solubility. In a deoxygenated

state, virtually all hemoglobin C cells have crystalloid inclusions.

Deoxygenation further reduces cell solubility and increases blood

viscosity. Add 3% salt solution to a drop of blood smear, and then the

crystals appear. Sickle-cell anemia is due to a mutation in the beta globin

chain of hemoglobin and the amino acid glutamic acid is replaced with

valine in the 6thposition. This lead to a mutation of the chain which

polymerizes under low oxygen tension causes the red cells to be distorted

and lose their elasticity. In fact the newer RBC are quite elastic and this

allows them to pass through the small size capillaries. However, with time

and low oxygen concentration, this elasticity is lost and the red cells

Page 36: USMLE Physiology Step 1 MCQ

maintain the deformed shape- sickle cell. These deformed cells are then

removed by the spleen. When the deformed cells plug up a blood vessel,

ischemic episodes (sickle cell crises) occur. F

7. A 22 year old male presents with a history of discolored urine

and vague abdominal pain. He says he has had these symptoms

for more than a year. It is decided that a Ham test should be done

to rule out?

a. Thalassemia

b. sickle cell

c. aplastic anemia

d. paroxysmal nocturnal hemoglobinuria

e. hemophilia

f. iron deficiency anemia

g. leukemia

Paroxysmal nocturnal hemoglobinuria (PNH) is a rare disorder which is

typically presents with aplastic anemia, thrombosis and discolored urine

(red). The diagnosis in the past was done with the so called HAM test.

Today, advances in flow cytometry can help assess presence of CD55,

CD16 and CD59 on both the white and red blood cells. PNH is then

subclassifed depending on the presence of these CD markers. D

8. In 22 year old with a prolonged labor stage, the obstetrician

decides to use a drug to thin, dilate and ripen the cervix. The

agent is

a. Finasteride

b. oxytocin

c. ergonovine

d. progesterone

e. dinoprostone

f. aspirin

g. nitroglycerine

Page 37: USMLE Physiology Step 1 MCQ

Dinoprostone works by causing the cervix to thin and dilate (open) and

the uterus to contract (cramp) the way it does during labor. E

9. A 65 yo female was seen in the clinic and brought her

abdominal x ray with her. She says that she has had a number of

episodic pain episodes in the RUQ associated with nausea and

vomiting. She kept on a fat free diet and subsequently she did

improve. She also has a radiology report which claims that she

has a rim of calcification around the entire gall bladder. You think

she may be at risk for?

a. Acalculous cholecystitis

b. Gall stone ileus

c. Cancer

d. Pancreatitis

e. Gall bladder fistula

f. Peptic ulcer disease

g. Duodenal perforation

h. Hepatitis

In some rare, the gall balder will form a calcified rim around it. This is

what is known as a porcelain gall bladder. There is some evidence that

porcelain gall gladder is a risk for cancer. Gall balder cancer is almost

spread before it is diagnosed. It generally spreads spread to nearby

organs and tissues such as the liver or small intestine. It also spreads

through the lymph system to lymph nodes in the region of the liver (portal

hepatis). Ultimately, other lymph nodes and organs can become involved.

The risk factors for gall bladder cancer are unknown. Although it occurs

most often in people with calcified or porcelain gall bladder-where

repeated inflammation from passing gallstones leads to hardening

(calcification) of the gall bladder, it is extremely rare even in such

patients. Since the gallbladder isn't essential, people with a calcified gall

bladder may consider having it removed as a preventative measure. C

10. A 2 year old thin male who smokes presents to the ER with

complaints of left sided chest pain. The pain came on suddenly

Page 38: USMLE Physiology Step 1 MCQ

and he acutely became short of breath. In the ER his pulse

oximetry is 94% at room air. He most likely may have developed?

a.Gastric perforation

b.Pneumothorax

c.CHF

d.Pneumonia

e.Asthma

f.Emphysema

Spontaneous pneumothorax: Although some view primary spontaneous

pneumothorax as more of a nuisance than a major health threat, deaths

have been reported. Secondary spontaneous pneumothorax can be life

threatening, depending on the severity of the underlying disease and the

size of the pneumothorax. Compared with similar patients without

pneumothorax, age-matched patients with chronic obstructive pulmonary

disease have a 3.5-fold increase in relative mortality when a spontaneous

pneumothorax occurs. Mortality percentages in patients with chronic

obstructive pulmonary disease and spontaneous pneumothorax vary from

1-17%.

For a complete review on MCQs on USMLE, contact

[email protected]

Posted by sbmedex at 8:31 PM 0 comments Links to this post

USMLE Step 1 Medicine & Surgery part 3

Please select one correct answer

1. A 35 yo is about to undergo an invasive medical procedure. She

tells you that she has Mitral valve prolapse. Which of the

following procedures does not require endocarditis prophylaxis?

a.elective C-section

b.upper endoscopy

c.cardiac catheterization

Page 39: USMLE Physiology Step 1 MCQ

d.dental extraction

e.appendectomy

f.Congenital valve surgery

Antibiotic prophylaxis is recommended for all invasive respiratory tract

procedures that involve incision or biopsy of the respiratory mucosa (e.g.,

tonsillectomy, adenoidectomy). Antibiotic prophylaxis is not

recommended for bronchoscopy unless the procedure involves a biopsy.

When anti biotic prophylaxis is used, it should cover Streptococcus

viridians.

Patients who are about to undergo a surgical procedure that involves

infected skin, skin structure, or musculoskeletal tissue, should receive an

agent active against staphylococci and beta-hemolytic streptococci (e.g.,

antistaphylococcal penicillin, cephalosporin).

If the causative organism of respiratory, skin, skin structure, or

musculoskeletal infection is known or suspected to be Staphylococcus

aureus, one should start a antistaphylococcal penicillin or cephalosporin,

or vancomycin (if patient unable to tolerate beta-lactam antibiotics).

Vancomycin is recommended for known or suspected Methicillin-resistant

strains of S aureus.

Antibiotics are no longer recommended for endocarditis prophylaxis for

patients undergoing genitourinary or gastrointestinal tract procedures.

2. A 59 yo is involved in a car wreck is brought to the ER

intubated. He is resuscitated and admitted to the ICU because of

severe brain injury. His chest x ray reveals that he has bilateral

fluffy infiltrates. Which of the following is the earliest finding of

ARDs within 12-24 hours after the onset of the disorder?

a.Fluffy infiltrates on chest x-ray

b.Hypercapnia

c.Extreme hypoxia

Page 40: USMLE Physiology Step 1 MCQ

d.Increased TV

e.Tachypnea

f.Fever

There are many physical signs that reflect lung pathology and other organ

injury associated with ARDS. However, the first physical sign of the

disorder is tachypnea. As pulmonary edema develops, the lung

compliance decreases and the tidal volume decreases towards the FRC

and the work of breathing increases. Patients will also become blue and

pale with time with increasing hypoxemia. Fever may be due to a ongoing

pneumonia, sepsis or may reflect a massive inflammatory process. All

patients with ARDs develop crackles in the lung fields. Other physical

signs may include the presence of an air leak, pneumothoraces,

pneumomediastinum, pneumopericardium, and subcutaneous

emphysema. As PEEP is increased the heart sounds will be muffled and

there will be signs of decreased cardiac output. E

3. A 71 yo requires mechanical ventilation because of respiratory

failure. Over the ensuing few days it is noticed that his

oxygenation is poor. The critical care physician decides to

increase the PEEP to 20 mmH20. The major effect of increasing

PEEP to such levels is?

a.Increasing venous return

b.Decreasing intracranial pressure

c.Increasing CO2 excretion

d.Decreasing PO2 levels

e.Decreasing cardiac output

The problem with PEEP is how much to give: insufficient PEEP is of little

benefit, excessive PEEP can cause three distinct problems:

Alveolar over distention in the upper part of the lungs is common.

Continued used of high PEEP can lead to barotrauma. Excessively high

alveolar pressures may narrow the capillaries which surround the

airspaces, causing an increase in dead space (wasted ventilation) and an

Page 41: USMLE Physiology Step 1 MCQ

unnecessary increase in the work of breathing.

Increased intrathoracic pressure as a result of PEEP will reduce the

pressure gradient along which blood returns to the heart (flow is always

from zones of high pressure to those of low pressure, the negative

intrathoracic pressure associated with inspiration enhances this effect).

This reduces right ventricular preload, right ventricular output and

ultimately cardiac output. This may lead to a reduction in blood pressure

and pooling of blood in the abdomen and peripheries. Conversely, in

severe heart failure this may be beneficial. E

4. The revised trauma score incorporates which of the following:

a.Patient age and chronic medical conditions

b.Glasgow coma scale, respiration rate and blood pressure

c.Physiologic and metabolism problems

d.Number of fractures

e.Co morbidity conditions

f.Eye, motor and visual problems

The Revised Trauma Score is made up of a combination of results from

three categories: Glasgow Coma Scale, Systolic Blood Pressure, and

respiratory rate. All of these results can be quickly assessed with minimal

equipment: a flashlight, a watch and a sphygmomanometer since systolic

pressure can be obtained through arterial palpation. The score range is 0-

12. with a score of 12, the patient is stable, 11 reflects urgent

(intervention is required but the patient can wait a short time), and 10-3 is

IMMEDIATE (immediate intervention is necessary). The last possible label

is MORGUE, which is given to seriously injured people with an RTS score of

3 or lower. B

5. A female has first degree burns to her entire left arm, second

degree burn to the front of the chest, abdomen and third degree

circumferential burn to her left thigh and lower leg. The fluid

requirements are what percent of the total body?

a.36%

Page 42: USMLE Physiology Step 1 MCQ

b.12%

c.72%

d.54%

e. 8%

Burns are judged by the size of the burn in relation to the whole body and

by the depth of the burn injury. Different methods exist to calculate the

extent or size of a burn injury. The most common method, which provides

a quick estimate of burn size, uses the "Rule of Nines," where the body is

divided into areas equaling multiples of 9% of the total body surface area.

The palm of your hand, for example, is equal to about 1% of your body's

surface area. The head and arms are each equal to 9% of the body

surface. The chest and back are each 18% (2 x 9%). Each leg is 18% (2 x

9%). This totals eleven nines, or 99%. The heads of infants and small

children are in relatively larger proportion to the total body surface area,

and the limbs are in relatively smaller proportion than adults limbs. The

total body surface area of a burn is referred to as TBSA, or total body

surface area. A patient might have the diagnoses of a 45% TBSA thermal

burn, for example. The TBSA and burn depth analysis are recorded on a

hospital chart known as a "burn diagram." Determining the percent of

body surface area burned is important for correct fluid resuscitation. A

6. A postoperative patient is given morphine for pain. Later he

develops nausea and is given an anti emetic. Soon he becomes

drowsy and has a respiration rate of 7/min. the next step in his

management is?

a. Send the patient to ICU

b. Observation

c. Dialysis to remove the anti emetic agent

d. Intubate and mechanically ventilation

e. Administer naloxone

f. Give 100% oxygen

g. Order arterial blood gas

h. Give an antidote to the anti emetic

Page 43: USMLE Physiology Step 1 MCQ

Naloxone is a drug used to counter the effects of opioid overdose, for

example heroin or morphine overdose. Naloxone is specifically used to

counteract life-threatening depression of the central nervous system and

respiratory system. It is marketed under various trademarks including

Narcan, Nalone, and Narcanti, and has sometimes been mistakenly called

"naltrexate." No time should be wasted ordering redundant chest x rays or

blood gas. E

7. An 18 yo steps on a dirty nail and develops pain in his forefoot.

In the ER, he is examined and it is suspected that he may have

acquired tetanus. The first sign of a tetanus infection is?

a.Dysphagia

b.Stiff neck

c.Convulsions

d.Muscle pain

e.Redness

f.Pain

g.Fever

The first signs of tetanus infection are usually a headache and spasms of

the jaw muscles. The victim may become irritable. As the poison spreads,

it causes muscle spasms in the neck, arms, legs, and stomach. The victim

may get painful convulsions, which can be severe enough to cause broken

bones. People with tetanus may have to spend several weeks in the

hospital under intensive care. In the United States, a tetanus infection

carries a mortality of 30%. B

Posted by sbmedex at 8:23 PM 0 comments Links to this post

Labels: USMLE Medicine

USMLE step 1: Anti TB drugs/Pharmacology

Please select the one correct answer

1. The principal first line drugs for Mycobacterium tuberculosis

include all except:

Page 44: USMLE Physiology Step 1 MCQ

a.Isoniazid

b.Ethambutol

c.Rifampin

d.Gentamycin

e.Pyrazinamide

All except Gentamycin are first line agents for tuberculosis because of

their efficacy and acceptable degrees of toxicity. D

2. Which of the following statement about Tuberculosis drugs is

false?

a.multi drug resistance is common

b.duration of treatment may range from 4-9 months

c.single drug therapy is effective for the majority of cases

d.side effects should be monitored

e.poor patient compliance also affects treatment

A minimum of two drugs should always be used for treatment and

sometimes even 3. This multi drug regimen is designed to prevent the

emergence of resistant strains. The regimen is continued until the clinical

disease has disappeared.

3. A 43 year old Asian is placed on Isoniazid for 4 months to treat

his TB. He comes to you with complains of painful tingling in his

arms and legs. The tingling has been getting intense on a daily

basis. The side effects of Isoniazid can be reversed with:

a.vitamin C

b.Niacin

c.Pyridoxine

d.Thiamine

e.Phenytoin

Peripheral neuritis is the most common adverse effect which is related to

Page 45: USMLE Physiology Step 1 MCQ

a relative pyridoxine deficiency. Most of the toxic reactions are corrected

by pyridoxine (vitamin B6) supplementation.

4. A patient with TB is treated on a multi-drug regimen. Four

weeks later he presents with complaints of red colored tears. The

most likely agent causing this is:

a. Isoniazid

b. Gentamycin

c. Rifampin

d. Ethambutol

e. Pyrazinamide

Urine and feces as well as tears can be orange colored when taking

Rifampin. C

5. A 43 year old patient is seen in the eye clinic with complaints

of eye pain. His vision is blurred and the retinal exam does reveal

mild neuritis. Because the patient is an immigrant, you suspect

that the cause of his Optic neuritis is due to:

a.Ethambutol

b.Pyrazinamide

c.Rifampin

d.Isoniazid

e.Gentamycin

Ethambutol can cause optic neuritis, which results in decreased visual

acuity and loss of ability to discriminate between red and green. Visual

acuity should be periodically examined. A

6. The drug of choice to treat (Leprosy) Hansen’s disease is:

a.Dapsone

b.Erythromycin

c.Isoniazid

Page 46: USMLE Physiology Step 1 MCQ

d.Gentamicin

Dapsone is structurally related to Sulfonamides. It is used also to treat

PCP. A

Posted by sbmedex at 8:15 PM 0 comments Links to this post

Labels: USMLE Pharmacology

USMLE Step 1 & 2- Trauma and Medicine MCQs

Please select one correct answer

1.A 71 year old is admitted to the CCU and needs invasive

monitoring. It is decided to place a swan Ganz catheter and

measure the pulmonary catheter wedge pressure. This will allow

one to assess the pressure in the?

a. Right atrium

b. Left atrium

c. Left ventricle

d. Right ventricle

e. Aorta

f. Inferior vena cava

Pulmonary capillary wedge pressure (PCWP) provides an indirect estimate

of left atrial pressure (LAP). Although left ventricular pressure can be

directly measured by placing a catheter into the left ventricle by feeding it

through a peripheral artery, into the aorta, and then into the ventricle, it is

not possible to place the catheter into the left atrium unless there was an

atrial septal defect present. The swan Ganz catheter is usually inserted at

the bedside under sterile conditions. It is frequently used to monitor

patients in the CCU and ICU. B

2. A 64 year old male in the ICU needs quick IV access. The

resident decides to insert a central venous line. Which is not a

complication of right internal jugular puncture?

Page 47: USMLE Physiology Step 1 MCQ

a. pneumothorax

b. hemomediastinum

c. thoracic duct injury

d. tracheal puncture

e. phrenic nerve injury

f. carotid artery dissection

g. injury to esophagus

Central line insertion can be associated with a number of complications.

However, the benefits derived from their benefits far overide the risks of

potential complications. The most common complication of central line

insertion is pneumothorax. There is a belief that the incidence of

pneumothorax is higher if the central line is attempted via the subclavain

vein. Many physicians utilize ultrasound to help identify the neck veins

before cannualtion. The reported incidece of penumothorax is about 1%

but this is unreported.

Other complications of central lines include infection chiefly from

Staphylococcus aureus and Staphylococcus epidermidis. The risk of

infection are high when the individual is diabetic, immunocomprmised and

obese. When a central line infection is suspected, the line should be

removed and the tip sent for culture. If the individual is febrile, blood

cultures should be obtained. No new line should be re inserted until the

cultures are negative or the fever has subsided. Other rare complications

of central lines include injury to the carotid artery, air embolism,

arrhythmias, pericardial tamponande and pain G

3.A 45 year old obese female undergoes a laparotomy for bowel

obstruction. The best way to manage her post operative pain is?

a. nurse controlled analgesia

b. pain medication prn

c. patient controlled analgesia

d. Epidural anesthesia

e. acupuncture

Page 48: USMLE Physiology Step 1 MCQ

Patient controlled analgesia is best for most individuals esp. children and

obese individuals. C

4. A 57 year old has been told that he has a mass behind the left

knee. Ultrasound reveals that it is a vascular mass. In general,

the most common presentation of a popliteal artery aneurysm

begins with?

a. pain

b. mass behind knee

c. decreased pulses

d. claudication

e. sudden onset of left cold leg

Popliteal artery aneurysms (PAAs) are the most common peripheral

arterial aneurysms and the second most common aneurysm after

abdominal aortic aneurysms. In the past, popliteal aneurysms have been

associated with a high rate of limb loss. When the aneurysm ruptures,

there is a sudden loss of blood supply to the distal leg. In many cases,

embolization to the distal blood vessels of the foot is also common.

Because of this awareness, all individuals with either asymptomatic or

symptomatic aneurysms of ≥ 2 cm in diameter should be considered for

elective pass surgery. While surgery was the standard of care in the past,

this has no changed with the availability of endovascular therapy.

Endovascular stenting is also an excellent option in the presence of a

ruptured aneurysm. Once a stent has been placed it is important that the

patient be placed in anticoagulation medications like ticlodipine. E

5. A 35 yo female has just delivered a baby and 2 days later is

found to have a swollen left leg. Ultrasound shows a deep vein

thrombus just beneath the popliteal vein. The next step in her

management is?

a. coumadin

b. heparin loading then infusion

Page 49: USMLE Physiology Step 1 MCQ

c. low molecular weight heparin

d. Aspirin

e. Green field filter

f. Compression stockings

g. ambulation

The primary aim for the treatment of DVT is to prevent pulmonary

embolism, reduce morbidity, and prevent or minimize the risk of

developing the postphlebitic syndrome. Anticoagulation is the mainstay of

the initial treatment for DVT. One may use either unfractionated heparin

or low molecular weight heparins to treat the DVT. Heparin does not lyse

the thrombus but prevents extension of thrombus. Heparin has been

shown to significantly reduce (but not eliminate) the incidence of fatal and

nonfatal pulmonary emboli as well as recurrent thrombosis. The chief

reason for the continuing risk of PE is mainly because heparin does not

affect the preexisting nonadherent thrombus. Heparin does not affect the

size of existing thrombus nor does it have any intrinsic thrombolytic

activity.

In less than 10% of patients is heparin therapy associated with complete

lysis. Heparin therapy has little effect on the risk of developing the

postphlebitic syndrome. The deep vein thrombus usually causes

destruction of the valves which almost always results in a high incidence

of chronic venous insufficiency and postphlebitic syndrome.

Warfarin therapy is usually started 3-5 days after heparin. The INR is

maintained anywhere between 2-2.5. Heparin must be overlapped with

oral warfarin because of the initial transient hypercoagulable state

induced by warfarin. This effect is related to the differential half-lives of

protein C, protein S, and the vitamin K–dependent clotting factors II, VII,

IX, and X. Long-term anticoagulation is definitely indicated for patients

with recurrent venous thrombosis and/or persistent or irreversible risk

factors. B

6. A 20 year old was involved in a motor vehicle accident. He had

severe blunt chest trauma to the chest. In the ER he received 8

Page 50: USMLE Physiology Step 1 MCQ

units of packed red blood cells. Soon after he became short of

breath and his PO2 was 55 on 100% oxygen. His chest x ray

revealed a complete white out and he had a Central venous

pressure of 11. At 48 hours after the injury, the nature of his lung

injury was most likely due to?

a. pulmonary contusion

b. ARDs

c. Pneumonia

d. PE

e. heart failure

f. lung fibrosis

Brusing or contusion of the lug is common after chest trauma. The

condition results in lng unjury hwhcih is often marked by fluid and blood

sccumualtion in th elung parenchyma. This results in difficutly in

oxygenation and hyperventilation. In lung contuison the lung tissue

structure is intact and there is no franck terar or rupture. Pulmonary lung

contuion rarely occurs in isolation iand is often associated with rib

fractures or soft tissue injury. The typical signs of lun contusion incldue

chest pain, fever, tachyonea and hemoptysis. The chest x ray may reveal

signs of a lung contuison, but a CT scan is required for a definitive

diagnosis. A

7. A 43 yo is involved in a car accident and breaks both femurs.

There is concern that he may develop fat embolism syndrome.

The radiographic findings of fat embolus are most typically

evidenced at what time period?

a. 0-12 hrs

b. 12-24 hrs

c. 24-48 hrs

d. 48-72 hrs

e. immediately

There is no radiological feature which is diagnostic of fat embolism

syndrome. Infact in the first few hours after injury, the chest x ray may be

Page 51: USMLE Physiology Step 1 MCQ

completely normal. Over the next 48 hours, some patients may develop

bilateral fluffy shadows which are associated with a decline in the

respiratory status. Some individuals may develop diffuse or patchy air

space consolidation, due to edema or alveolar hemorrhage; this is most

prominent in the periphery and bases. Radiological studies like the

Ventilation/perfusion scans may demonstrate a mottled pattern of sub-

segmental perfusion defects with a normal ventilator pattern. In most

cases, the earliest features on x ray may be seen around 48-72 hours.

CT may identify ill defined areas of centrilobular and sub pleural nodules

representing alveolar edema, microhemorrhage, and inflammatory

response secondary to ischemia and cytotoxic emboli may be seen. MRI of

the brain may reveal high-intensity T2 signals; this correlates with the

degree of neurological impairment found clinically.

Fat particles are not present in more than 50% of individuals with fat

embolism syndrome. There are some who advocate the use of

bronchoscopy with bronchoalveolar lavage to detect fat droplets in

alveolar macrophages as a means to diagnose fat embolism has been

described in trauma patients and patients with the acute chest syndrome

of sickle cell disease. However, diagnostic criteria vary and the sensitivity

and specificity are unknown. D

Posted by sbmedex at 6:55 PM 0 comments Links to this post

Labels: USMLE Trauma

USMLE Step 1 and 2, Medicine and Pharmacology

Please select one correct answer

1.A 30 yo black female presents with a 4 month history of a dry

cough and general malaise. She does not smoke and her TB skin

test was negative. She did have an episode of renal colic 2 weeks

ago. Her x ray shows some mediastinal widening near the hilum

area. She may have?

a.HIV

b.Lobar pneumonia

c.Churg Strauss

Page 52: USMLE Physiology Step 1 MCQ

d.Sarcoidosis

e.Pericardial tamponade

f.Pneumothorax

g.PCP

h.Aortic dissection

Sarcoidosis is a disorder of unknown etiology. It tends to occur more

commonly in black females and is characterized by non-caseating

granulomas. The disease may only affect the lung but can be widespread

and affect the entire body. Most people have no symptoms. Sarcoid

typically presents on X ray as mediastinal adenopathy and needs to be

differentiated from TB. The classic symptoms of sarcoid include loss of

energy, lethargy, generalized myalgias and arthralgia, dry and burning

eyes, shortness of breath, a dry cough etc. some individuals may develop

erythema nodosum or a skin rash. Blood work in individuals with

sarcoidosis will reveal hypercalcemia and elevated vitamin D levels. When

sarcoidosis presents with erythema nodosum, bilateral hilar adenopathy

and arthralgia, this is known as Lofgren syndrome. Most patients have a

good prognosis after the diagnosis. Sarcoid is essentially a systemic

disorder and can affect the eye which can cause blindness. When the

parotid gland and eye are affected in individuals with sarcoidosis, this is

known as Heerfordt-Waldenstrom syndrome.

2. A 22 year old is admitted because of a coagulopathy. He says

he has a tendency to bleed and his brother died from the

disorder. Blood work reveals a PT of 12.4 s, PTT of 38 s, INR 1.2,

platelets 311 and a normal bleeding time. The platelet

aggregation test reveals lack of GP2B/3A. Of the following

disorders he most likely has

a.Bernard Soulier

b.hemophilia

c.protein C deficiency

d.glanzmann’s thrombasthenia

e.von willebrand’s

Page 53: USMLE Physiology Step 1 MCQ

Glanzmann's thrombasthenia is a very rare disorder and acquired via an

autosomal recessive trait. The platelets formed lack the GP2B/3A

receptor, thus preventing the platelet from aggregating together. Despite

normal level of platelets the bleeding time is prolonged. D

3. In which coagulopathy will one see a significant level of d

dimers?

a.hypothermia

b.DIC

c.MI

d.Bernard soulier

e.use of Ticlid

Disseminated intravascular coagulation (DIC) is a pathological process

where by the blood coagulation system goes havoc. The coagulation

cascade is activated continuously and the platelets are used up. The

disorder typically occurs in sick and critically ill patients. The usual cause

is an infection- particular gram negative sepsis. The blood work usually

reveals low levels of platelets, and elevated levels of fibrin degradation

products and D dimer. The bleeding time is prolonged. DIC- d dimers B

4. A 55 year old is found to have the translocation of the Bcr/abl

gene. The most likely defect he has is

a.Hodgkin’s

b.CML

c.CLL

d.burkitt’s

e.non Hodgkin’s

The Philadelphia chromosome involves translocation between

chromosome 9 and 22. The Bcr/abl gene is involved in this translocation.

The Philadelphia chromosome is commonly seen in chronic myelogenous

leukemia. The complete absence of the Philadelphia chromosome

indicates a very poor prognosis. The Philadelphia chromosome is also

Page 54: USMLE Physiology Step 1 MCQ

seen in ALL and AML but to a lower degree. B

5. A 59 yo is admitted with chest pains. He is started in an IV NTG

drip. The rapid heart rate sometimes seen after nitroglycerin

administration is best explained by:

a. direct effect on the heart

b. reflex sympathetic discharge due to a fall in systemic blood pressure

c. due to release of norepinephrine from sympathetic nerve endings

d. inhibition of cholinergic activity on the heart

Nitroglycerine decreases blood pressure and causes a reflex tachycardia.

B

6 Digoxin is useful in atrial fibrillation because it:

a. slows the sinus node activity

b. stimulates cholinergic activity

c. slow conduction through the A-V node

d. blocks the norepinephrine response to the heart

e. decreases the rate of conduction through atrial muscle

Digoxin acts at the AV node. C

7. When digitalis therapy is initiated, serious cardiac arrhythmias

may be caused by a deficiency of:

a. Sodium

b. Potassium

c. Chloride

d. Bicarbonate

e. Uranium

Potassium and magnesium deficiency are both known to worsen digoxin

toxicity. B

Page 55: USMLE Physiology Step 1 MCQ

8 Digitalis toxicity manifested by premature ventricular

contractions may

be treated with all of the following EXCEPT:

a. Lidocaine

b. Digitalis-specific immune FAB antibody

c. Phenytoin

d. Quinidine

e. Potassium replacement

Lidocaine is of no benefit in treatment of digoxin toxicity.

Posted by sbmedex at 6:34 PM 0 comments Links to this post

Labels: USMLE Step 1 Review 1

USMLE Step 1- Physiology

Please select the one correct answer

1. Which of the following is false about bombesin?

a. it is an autocrine growth factor

b. it is found in the gi tract

c. contracts gall bladder

d. releases pancreatic enzymes

e. functions as an endocrine hormone

A peptide that is found in the intrinsic nerves of the gastrointestinal tract,

bombesin stimulates the release of gastrin and pancreatic enzymes and

causes contraction of the gallbladder. These functions may be secondary,

however, to the release of cholecystokinin, a hormone secreted by the

mucosa of the intestine that has similar effects. E

2. Which lung cancer is commonly associated with hypercalcemia?

a. small cell

b. squamous cell

Page 56: USMLE Physiology Step 1 MCQ

c. adenocarcinoma

d. carcinoid

e. bronchoalveolar

Humoral hypercalcemia of malignancy, resulting from the production of

parathyroid hormone-related protein by the tumor, is most commonly

associated with squamous cell carcinoma. B

3. Which lung cancer is most commonly associated with the

SIADH SYNDROME?

a. Squamous cell

b. Carcinoid

c. Small cell

d. Adenocarcinoma

The SIADH is also more common in small cell carcinoma, occurring in 7%

to 11% of patients. The manifestations of hyponatremia (mental status

changes, lethargy, or seizures) are often absent despite very low sodium

levels, as the rate of decline is typically prolonged. C

4. In ethanol drinking, the typical enzymatic pattern of liver

enzymes is?

a. AST> ALT

b. ALT> AST

c. elevated alkaline phosphatase

d. elevated bilirubin

Diagnosing ALD is a challenge. A history of heavy alcohol use along with

certain physical signs and positive laboratory tests for liver disease are

the best indicators of disease. Alcohol dependence is not necessarily a

prerequisite for ALD, and ALD can be difficult to diagnose because

patients often minimize or deny their alcohol abuse. Even more

confounding is the fact that physical exams and lab findings may not

specifically point to ALD. Diagnosis typically relies on laboratory tests of

Page 57: USMLE Physiology Step 1 MCQ

three liver enzymes: gamma–glutamyltransferase (GGT), aspartate

aminotransferase (AST), and alanine aminotransferase (ALT). Liver

disease is the most likely diagnosis if the AST level is more than twice that

of ALT, a ratio some studies have found in more than 80 percent of

alcoholic liver disease patients. An elevated level of the liver enzyme GGT

is another gauge of heavy alcohol use and liver injury. Of the three

enzymes, GGT is the best indicator of excessive alcohol consumption, but

GGT is present in many organs and is increased by other drugs as well, so

high GGT levels do not necessarily mean the patient is abusing alcohol. A

5. Parafollicular cells of the thyroid secrete:

a. Calcitonin

b. Amyloid

c. Thryoglobulin

d. Parathyroid hormone

e. Iodine

Parafollicular cells also called C cells, are cells in the thyroid which

produce and secrete calcitonin. A

For more USMLE Questions, write to [email protected].

500 questions for $10.

Posted by sbmedex at 4:44 PM 0 comments Links to this post

Labels: USMLE MCQs Physiology

S U N D A Y , F E B R U A R Y 8 , 2 0 0 9

USMLE Review step 1 MCQs

1. A 10 year old girl is seen in the neurology clinic with

complaints of severe burning pain in her extremities and ongoing

fever that has been going on for years. Her mother says that the

physician told her it was a congenital problem. Examination also

reveals the presence of numerous dark skin lesions over the

lower trunk. Biopsy of the lesions reveals an excess of ceramide.

Page 58: USMLE Physiology Step 1 MCQ

She most likely has:

a. gaucher’s

b. tay sachs

c. lesch nynn

d. fabrys

e. hurler’s

X-Linked Recessive. alpha-Galactosidase A deficiency leads to a buildup of

ceramide trihexoside in body tissues.. Angiokeratomas (skin lesions) occur

in the lower trunk, fever, severe burning pain in extremities. Fabry's

disease is also be associated with involvement of the cardiac and brain

organs. Other symptoms include decreased sweating, tiredness and

angiokeratomas.

Angiokeratomas are small, painless skin lesions that may appear on any

region of the body, but are most common on the legs, abdomen and groin

area. Often the eyes are involved and causes keratopathy. other eye

lesions include cataracts, papilledema, optic atrophy and mecualr

swelling. D

2. Deficiency of adenosine deaminase is seen in?

a. AIDs

b. SCIDs

c. DiGeorge

d. CGD

e. Jobs

Autosomal Recessive. Adenosine Deaminase deficiency leads to

accumulation of dATP. This leads to a decrease in DNA precursors

molecules. Severe deficiency in both humoral and cellular immunity

occurs due to impaired DNA synthesis. Bone marrow transplant may be

helpful in treatment. B

3. The most common type of immune deficiency in children is?

Page 59: USMLE Physiology Step 1 MCQ

a. AIDs

b. DiGeorge

c. Selective IgA

d. SCID

e. CGD

The most common congenital immune deficiency is selective IgA. There

also exists selective IgM and IgG deficiencies, but they are less common.

IgA deficiency may be due to a failure of heavy-chain gene switching.

4. A 43 yo female with a dilated cardiomyopathy suddenly

developed this rhythm. The patient has a BP of 50/10 and the

pulse is very weak. You are on call to the Medical Floor and the

nurse asks you to do something quick. Please look at the

attached link and decide that the best treatment is?

a. beta blocker

b. quinidine

c. amiodarone

d. digoxin

e. Defibrillation

Amiodarone is an anti arrhythmic drug used for various types of

tachyarrhythmias, both ventricular and supraventricular (atrial)

arrhythmias. it is a superb drug which can immediately reverse atrial

fibrillation and is also helpful for ventricular tachycardias. However, in any

patient with low blood pressure and a rhythm which is shown (VF), urgent

defibrillation is required. The first treatment of choice in a patient with

ventricular tachycardia is defibrillation at 360 joules. E

5. A 19 yo with no symptoms had this ECG before a work out. He

most likely has:

a. first degree HBb.

b. atrial flutter

Page 60: USMLE Physiology Step 1 MCQ

c. Normal sinus rhythm

d. atrial fibrillation

e. ventricular arrhythmia

f. torsades de pointes

This is normal sinus rhythm. C

6. A 43 yo with mitral stenosis is found to have a dilated left

atrium. His rhythm strip is shown below. He most likely has?

a. atrial flutter

b. premature atrial beats

c. first degree heart block

d. atrial fibrillation

e. ventricular tachycardia

f. sinus tachycardia

Atrial fibrillation is common with advancing age; however other causes

include uncontrolled hypertension, coronary disease, CHF, valvular heart

disease, acute pulmonary processes, hyperthyroidism, acute alcohol

intoxication . Patients having undergone cardiac, pulmonary, or

esophageal surgery have a 20-40% postoperative incidence of atrial

fibrillation (AF). Patients with congenital heart disease are at higher risk

for developing AF. The most common arrhythmia seen in patients with

mitral stenosis is atrial fibrillation. The dilated atrium also has a tendency

to form a clot which frequently embolizes. if you see an ECG without any P

waves, irregular heart rate with narrow QRS, it is Atrial fibrillation until

proven otherwise. D

Posted by sbmedex at 7:18 AM 0 comments Links to this post

Labels: Medicine, Pharmacology, Physiology, USMLE MCQs

S A T U R D A Y , F E B R U A R Y 7 , 2 0 0 9

USMLE Step 1 Review (a)

1. An angiogram reveals stenosis of the right innominate artery.

The artery can best be exposed by a?

Page 61: USMLE Physiology Step 1 MCQ

a. right thoracotomy

b. right supraclavicular incision

c. median sternotomy

d. neck incision

Innominate artery is best exposed by a median sternotomy. the right

innominate artery gives off the right subclavian and the right common

carotid artery. C

2. A patient with a traumatic aortic rupture undergoes aortic

repair. The patient presents with hoarseness after surgery. The

most likely cause of hoarseness is?

a. postoperative viral infection

b. damage to phrenic nerve

c. trauma to recurrent laryngeal nerve

d. injury to sympathetic chain

e. damage to vocal cords during intubation

Hoarseness is due to injury to the recurrent laryngeal nerve. The recurrent

nerve runs in between the left common carotid and the left subclavian

and can easily be injured by placing a clamp in that region. C

3. A 35 year old male is involved in a head on motor vehicle

accident. On arrival to the emergency room, he has severe

bruises to his chest but is otherwise stable. The chest X ray

reveals a widen mediastinum. An angiogram is ordered which

reveals a traumatic aortic rupture. The most common site of

aortic rupture after severe blunt trauma to the chest is ?

a. ascending aorta

b. abdominal aorta

c. aortic arch

d. descending aorta distal to left subclavian artery

Page 62: USMLE Physiology Step 1 MCQ

e. femoral artery

f. left common carotid artery

Following blunt trauma, the descending aorta just distal to the left

subclavian artery is most prone to injury. A chest x ray will reveal a widen

mediastinum and a CT scan is required for diagnosis. D

4. A new born is seen in the infectious disease clinic due to

repeated candida infections. Blood work reveals that he has very

low levels of calcium and has suffered from numerous tetany

spells. He most likely has:

a. AIDs

b. SCIDs

c. DiGeorge

d. CGD

e. renal failure

T-Cell deficiency occurs when the thymus is absent. Hypocalcemic tetany

occurs from primary parathyroid deficiency. In Digeorge syndrome, the

parathyroid and thymus glands are missing. C

5. Failure to develop the 3 and 4th pharyngeal pouches are linked

to what immune disorder?

a. AIDs

b. SCIDs

c. DiGeorge

d. CGD

e. Job’s

Failure of development of the 3rd and 4th Pharyngeal Pouches leads to

agenesis of the thymus and parathyroid glands. C

Posted by sbmedex at 8:40 PM 0 comments Links to this post

Page 63: USMLE Physiology Step 1 MCQ

Labels: AIDs, DiGeorge syndrome, parathyroid, SCIDs, thymus

USMLE Step 1 Review

1. The first branch of the aorta is?

a. right brachiocephalic artery

b. left common carotid

c. left subclavian artery

d. right coronary artery

e. right common carotid artery

The right coronary is the first branch of the aorta. D

2. A patient has been involved in a MVA. He presents to the

emergency room with severe pain in his right chest. The chest x

ray reveals numerous rib fractures and pulmonary contusion.

Which finding on the physical examination would make you

suspect that would make you suspect flail chest is?

a. difficulty breathing

b. increased stridor

c. paradoxical breathing

d. low oxygen saturation

e. pain

f. hyperventilation

Paradoxical breathing is a sin que non of flail chest. These patients need

arterial blood gas monitoring, pain control and pulmonary care. C

3. The main reason a tracheostomy is not performed on the first

cartilage ring is because:

a. trachea is too narrow

b. a high chance of subglottic stenosis

c. a high chance of tracheo-innominate artery fistula

Page 64: USMLE Physiology Step 1 MCQ

d. inability to access it

e. technically impossible

f. chance of damaging esophagus

Tracheostomy performed in the first cartilage ring can result in subglottic

stenosis, which is almost impossible to repair. B

4. The most common mass in the anterior mediastinum is ?

a. thymoma

b. thyroid

c. Teratoma

d. lymphoma

e. leukemia

f. seminoma

Thymoma is the most common mass of the anterior mediastinum. A

5. The left lateral border and apex of the cardiac silhouette is

made up by the

a. right atrium

b. right ventricle

c. left atrium

d. left ventricle

The left ventricle makes up the major border of the left cardiac silhouette.

D

Posted by sbmedex at 8:32 PM 0 comments Links to this post

Labels: USMLE Step 1 Review 1

PASS the USMLE: USMLE Surgery MCQs Step 1 & 2 (1)

PASS the USMLE: USMLE Surgery MCQs Step 1 & 2 (1)

Posted by sbmedex at 12:43 PM 0 comments Links to this post

Page 65: USMLE Physiology Step 1 MCQ

USMLE Surgery MCQs Step 1 & 2 (1)

1. A postoperative patient is given morphine for pain. Later he

develops nausea and is given an anti emetic. Soon he becomes

drowsy and has a respiration rate of 7/min. the next step in his

management is?

a. send the patient to ICU

b. observation

c. dialysis to remove the anti emetic agent

d. intubate and mechanically ventilation

e. administer naloxone

f. give 100% oxygen

g. order arterial blood gas

i. give an antidote to the anti emetic

Naloxone is a drug used to counter the effects of opioid overdose, for

example heroin or morphine overdose. Naloxone is specifically used to

counteract life-threatening depression of the central nervous system and

respiratory system. It is marketed under various trademarks including

Narcan, Nalone, and Narcanti, and has sometimes been mistakenly called

"naltrexate." No time should be wasted ordering redundant chest x rays or

blood gas. E

2. An 18 yo steps on a dirty nail and develops pain in his forefoot.

In the ER, he is examined and it is suspected that he may have

acquired tetanus. The first sign of a tetanus infection is?

a. dysphagia

b. stiff neck

c. convulsions

d. muscle pain

e. redness

f. pain

g. fever

Page 66: USMLE Physiology Step 1 MCQ

The first signs of tetanus infection are usually a headache and spasms of

the jaw muscles. The victim may become irritable. As the poison spreads,

it causes muscle spasms in the neck, arms, legs, and stomach. The victim

may get painful convulsions, which can be severe enough to cause broken

bones. People with tetanus may have to spend several weeks in the

hospital under intensive care. In the United States, a tetanus infection

carries a mortality of 30%. B

3. A 33 yo male with HIV presents with confusion and general

malaise. Blood work reveals that he is anemic and a CT scan

shows that he has generalized brain atrophy. The most common

type of HIV disorder of the brain is?

a. myelopathy

b. aseptic meningitis

c. AIDs dementia complex

d. Peripheral neuropathy

e. Encephalitis

f. Nocardia infection

g. cryptococcus

AIDS dementia complex (ADC; also known as HIV dementia, HIV

encephalopathy is a common neurological disorder associated with HIV

infection and AIDS. It is believed to be a metabolic encephalopathy

induced by HIV infection and involves interaction of the immune system.

These infected brain cells secrete a variety of neurotoxins which continue

to worsen the situation. The essential features of ADC are disabling

cognitive impairment accompanied by motor defects, improper speech

problems and a change in behavior. Most of the individuals show poor

memory, inability to concentrate and have mental clouding. The Motor

symptoms may include a loss of fine motor control which eventually leads

to inability to hold things, poor balance and recurrent tremors. Behavioral

alterations are significant and often include apathy, lethargy and flat

emotional responses and no spontaneity. Histopathologically, the brain is

infiltrated with monocytes and macrophages into the central nervous

system (CNS). there is gliosis, pallor of myelin sheaths, abnormalities of

Page 67: USMLE Physiology Step 1 MCQ

dendritic processes and neuronal apoptosis. ADC typically occurs after

many years of HIV infection and is associated with low CD4+ T cell levels

and high plasma viral loads. It is sometimes seen as the first sign of the

onset of AIDS. Prevalence is between 10-20% in Western countries.

4. A 43 year old male is seen in the anesthesia clinic before his

Lasik procedure. However, he does mention during the

preoperative assessment that he has been passing bright red

blood per rectum. The next step in his management is?

a. complete the Lasik surgery and the refer to gastroenterology

b. remove the hemorrhoids

c. cancel the Lasik surgery and order a upper endoscopy

d. cancel surgery and order a colonoscopy

e. perform a rectal exam

f. do the Lasik and hemorrhoid surgery at the same time

Lasik is an elective procedure. Anytime an individual has another medical

problem, the elective surgery should be canceled. This 43 year old male

may have hemorrhoids but bright red blood per rectum should always be

investigated. In this age group, a rectal exam is a must. In the older age

group, a colonoscopy or a barium enema is highly recommended E.

5. A 54 year old undergoes a laparotomy for bowel obstruction.

The surgery is uneventful and only requires resection of an

adhesive band. Postoperatively on the first day the patient

develops a fever. The fever is most likely due to:

a. atelectasis

b. abscess collection

c. pneumonia

d. urinary tract infection

e. Deep vein thrombosis

f. Drug related

The most common cause of early fever post operatively is atelectasis. The

Page 68: USMLE Physiology Step 1 MCQ

primary complication of atelectasis is hypoxemia, which is usually

transient. Within 24-48 hours, the lung area collapses and fails to be

ventilated. This is probably caused by a whole host of chemicals like

serotonin, histamine, prostaglandins, etc.The associated lung collapse is

always followed by profound vasoconstriction in the lung. If the atelectasis

is severe, it may cause enough hypoxemia acutely to require

supplemental oxygen or inhaler/ventilatory support.

Atelectasis is a suggested cause of fever in the early postoperative period.

However, all fever early in the post operative period is not always due to

atelectasis. Patients with temperatures of more than 38.5°C were less

likely to have atelectasis on radiography findings than those patients who

were afebrile and undergoing radiography as part of the postoperative

routine. A

Posted by sbmedex at 12:20 PM 0 comments Links to this post

Labels: 1, USMLE surgery

USMLE Surgery MCQs- Step 1 & 2

1. A 35 yo is about to undergo an invasive medical procedure. She

tells you that she has Mitral valve prolapse. Which of the

following procedures does not require endocarditis prophylaxis?

a. elective C-section

b. upper endoscopy

c. cardiac catheterization

d. dental extraction

e. appendectomy

f. congenital valve surgery

Antibiotic prophylaxis is recommended for all invasive respiratory tract

procedures that involve incision or biopsy of the respiratory mucosa

(e.g.,tonsillectomy, adenoidectomy). Antibiotic prophylaxis is not

recommended for bronchoscopy unless the procedure involves a biopsy.

When anti biotic prophylaxis is used, it should cover Streptococcus

viridans.

Page 69: USMLE Physiology Step 1 MCQ

Patients who are about to undergo a surgical procedure that involves

infected skin, skin structure, or musculoskeletal tissue, should receive an

agent active against staphylococci and beta-hemolytic streptococci (e.g.,

antistaphylococcal penicillin, cephalosporin). If the causative organism of

respiratory, skin, skin structure, or musculoskeletal infection is known or

suspected to be Staphylococcus aureus, one should start a

antistaphylococcal penicillin or cephalosporin, or vancomycin (if patient

unable to tolerate beta-lactam antibiotics). Vancomycin is recommended

for known or suspected Methicillin-resistant strains of S aureus. Antibiotics

are no longer recommended for endocarditis prophylaxis for patients

undergoing genitourinary or gastrointestinal tract procedures. B

2. A 59 yo is involved in a car wreck is brought to the ER

intubated. He is resuscitated and admitted to the ICU because of

severe brain injury. His chest x ray reveals that he has bilateral

fluffy infiltrates. Which of the following is the earliest finding of

ARDs within 12-24 hours after the onset of the disorder?

a. fluffy infiltrates on chest x-ray

b. hypercapnia

c. extreme hypoxia

d. increased TV

e. Tachypnea

f. fever

There are many physical signs that reflect lung pathology and other organ

injury associated with ARDS. However, the first physical sign of the

disorder is tachypnea. As pulmonary edema develops, the lung

compliance decreases and the tidal volume decreases towards the FRC

and the work of breathing increases. Patients will also become blue and

pale with time with increasing hypoxemia. Fever may be due to a ongoing

pneumonia, sepsis or may reflect a massive inflammatory process. All

patients with ARDs develop crackles in the lung fields. Other physical

signs may include the presence of an air leak, pneumothoraces,

pneumomediastinum, pneumopericardium, and subcutaneous

Page 70: USMLE Physiology Step 1 MCQ

emphysema. As PEEP is increased the heart sounds will be muffled and

there will be signs of decreased cardiac output. E

3. A 71 yo requires mechanical ventilation because of respiratory

failure. Over the ensuing few days it is noticed that his

oxygenation is poor. The critical care physician decides to

increase the PEEP to 20 mmH20. The major effect of increasing

PEEP to such levels is?

a. Increasing venous return

b. Decreasing intracranial pressure

c. Increasing CO2 excretion

d. Decreasing PO2 levels

e. Decreasing cardiac output

The problem with PEEP is how much to give: insufficient PEEP is of little

benefit, excessive PEEP can cause three distinct problems:

1. Alveolar over distention in the upper part of the lungs is common.

Continued used of high PEEP can lead to barotrauma.

2. Excessively high alveolar pressures may narrow the capillaries which

surround the airspaces, causing an increase in dead space (wasted

ventilation) and an unnecessary increase in the work of breathing.

3. Increased intrathoracic pressure as a result of PEEP will reduce the

pressure gradient along which blood returns to the heart (flow is always

from zones of high pressure to those of low pressure, the negative

intrathoracic pressure associated with inspiration enhances this effect).

This reduces right ventricular preload, right ventricular output and

ultimately cardiac output. This may lead to a reduction in blood pressure

and pooling of blood in the abdomen and peripheries. Conversely, in

severe heart failure this may be beneficial. E

4. The revised trauma score incorporates which of the following:

a. patient age and chronic medical conditions

b. Glasgow coma scale, respiration rate and blood pressure

c. Physiologic and metabolism problems

Page 71: USMLE Physiology Step 1 MCQ

d. Number of fractures

e. Co morbidity conditions

f. Eye, motor and visual problems

The Revised Trauma Score is made up of a combination of results from

three categories: Glasgow Coma Scale, Systolic Blood Pressure, and

respiratory rate. All of these results can be quickly assessed with minimal

equipment: a flashlight, a watch and a sphygmomanometer since systolic

pressure can be obtained through arterial palpation. The score range is 0-

12. with a score of 12, the patient is stable, 11 reflects urgent

(intervention is required but the patient can wait a short time), and 10-3 is

IMMEDIATE (immediate intervention is necessary). The last possible label

is MORGUE, which is given to seriously injured people with an RTS score of

3 or lower. B

5. A female has first degree burns to her entire left arm, second

degree burn to the front of the chest, abdomen and third degree

circumferential burn to her left thigh and lower leg. The fluid

requirements are what percent of the total body?

a. 36%

b. 12%

c. 72%

d. 54%

e. 8%

Burns are judged by the size of the burn in relation to the whole body and

by the depth of the burn injury. Different methods exist to calculate the

extent or size of a burn injury. The most common method, which provides

a quick estimate of burn size, uses the "Rule of Nines," where the body is

divided into areas equaling multiples of 9% of the total body surface area.

The palm of your hand, for example, is equal to about 1% of your body's

surface area. The head and arms are each equal to 9% of the body

surface. The chest and back are each 18% (2 x 9%). Each leg is 18% (2 x

9%). This totals eleven nines, or 99%. The heads of infants and small

children are in relatively larger proportion to the total body surface area,

Page 72: USMLE Physiology Step 1 MCQ

and the limbs are in relatively smaller proportion than adults limbs. The

total body surface area of a burn is referred to as TBSA, or total body

surface area. A patient might have the diagnoses of a 45% TBSA thermal

burn, for example. The TBSA and burn depth analysis are recorded on a

hospital chart known as a "burn diagram." Determining the percent of

body surface area burned is important for correct fluid resuscitation. A